LSAT and Law School Admissions Forum

Get expert LSAT preparation and law school admissions advice from PowerScore Test Preparation.

 Administrator
PowerScore Staff
  • PowerScore Staff
  • Posts: 8916
  • Joined: Feb 02, 2011
|
#26782
(The complete setup for this game can be found here: lsat/viewtopic.php?t=11235)

If Y is out then R must be in: R out brings T out with it, and that, along with Y, is 3 people out (too many). With R in you have OR in 1-2; M must be out (see above) with Y; L or T must be in 3, and then the other of L or T must be in 4. Remember that L and T can possibly go together. Here L in 4 is the "could" answer.
 VamosRafa19
  • Posts: 39
  • Joined: Nov 14, 2020
|
#82610
Administrator wrote: Wed Jun 29, 2016 10:01 am (The complete setup for this game can be found here: lsat/viewtopic.php?t=11235)

If Y is out then R must be in: R out brings T out with it, and that, along with Y, is 3 people out (too many). With R in you have OR in 1-2, M out (see above) with Y, L or T in 3, and then L, T, or X in 4. Remember that L and T can possibly go together. Here L in 4 is the "could" answer.
Is this correct? It says Y, L, or T in three but per rule 1 only L or T can be in 3.
 Adam Tyson
PowerScore Staff
  • PowerScore Staff
  • Posts: 5153
  • Joined: Apr 14, 2011
|
#82768
I can see how that answer might have been confusing the way it was written, VamosRafa19! I've changed the punctuation to make clear that what we were saying was "M must be out (along) with Y" and, apart from that, "L or T must be in 3". So you were absolutely correct, and that is what meant. Good eye!

Get the most out of your LSAT Prep Plus subscription.

Analyze and track your performance with our Testing and Analytics Package.